Está en la página 1de 12

Estonian math

ompetitions 2000/2001 2. Find the largest real number K having the following property: for any positive
real numbers a; b; satisfying the inequality a b 6 K , the inequality ab 6 K
also holds.
+ +

We thank the IMO ommunity for many of these problems whi h have been taken
from various materials distributed at the re ent IMO-s. 3. Prove that, for any integer n > , the number | {z: : : } is divisible by , but is
0 11 1 3
n

3 digits n

not divisible by +1 . 3
n

Autumn Open Contest: O tober 2000


4. The terms of the sequen e a1; a2; a3; : : : satisfy the ondition a a 1 a 2
for any n > . Find the sum of the rst terms of this sequen e, if the sum
n = n n

of the rst terms is and the sum of the rst terms is .


3 2000
Juniors (up to 10th grade)
1997 2002 2002 1997

1. How many positive integers less than and not ontaining digits other 5. On a plane n points are given, no three of them ollinear. At most how many line
than and are there? segments it is possible to draw between these points in su h a way that the line
20002001

segments form no triangle with verti es at the given points?


0 2

2. Find the two last digits of the number :::


1! + 2! + 3! + .
+ 2000!

3. Consider points C1 , C2 on the side AB of a triangle ABC , points A1 , A2 on Solutions of Autumn Open Contest
the side BC and points B1 , B2 on the side CA su h that these points divide the
orresponding sides to three equal parts. It is known that all the points A1 , A2 , J1. .
B1 , B2 , C1 and C2 are on y li . Prove that triangle ABC is equilateral. Answer: 136

The set of integers under onsideration onsists of all integers with up to digits
4. Real numbers x and y satisfy the system of equations ontaining only digits and , all1 -digit integers of the form  and the3
7

integer . There are integers with exa tly k digits and , and
0 2 8 20000
k

integers of the form  . So the required number of integers is


8 20002000 2 0 2 2
> x
<x+y +
>
> = 10
20000

y 0 + 21 + : : : + 26) + 8 + 1 = (27
: (2 1) + 9 = 136 :
>
> x(x + y)
>
:
y
= 20
J2. Answer: 13 .
Find the sum of all possible values of the expression x y . The produ t   : : :  has , and as fa tors, therefore being divisible by
. Hen e the last two digits of n are zeros for any n > and it su es to
1 2 10 2 5 10
+

nd two last digits of . The two last digits of the summands are
100 ! 10

5. Let m m for any odd integer m and m m for any even integer m .
= +3 =
01 02 06 24 20
:::
, , , , , , , and , yielding as the answer.
20
1! + 2! +

40 20
+ 9!

80 13

a) Find all integers k su h that k .


2

b) Prove that, for every odd integer K , there exist pre isely three dierent
= 1

C
integers k su h that k K .
) How many dierent integers k with the property k K exist for an even
=
B1 A2
integer K ?
= r r

B2 r
A1
r

r r

Seniors (grades 11 and 12) A C1 C2 B


Figure 1
1. Points A , B , C , D , E and F are given on a ir le in su h a way that the three
hords AB , CD and EF interse t in one point. Express angle EF A in terms of J3. Label the points on the sides of the triangle so that jAC1j jC1C2j jC2Bj ,
angles ABC and CDE (nd all possibilities). jBA1 j jA1 A2j jA2 C j and jCB1 j jB1 B2 j jB2 Aj (see Fig. 1). Then we have
= =

= = = =

1 2
6 BA1 C2 = 6 BA2 C1 = 6 BCA and 6 BC2A1 = 6 BC1 A2 = 6 BAC . Sin e points In ases (a) and (b) similar arguments give 6 EF A 6 CDE 6 ABC and
A1 , A2 , C1 and C2 are on y li , we get 6 BA2 C1 = 180Æ 6 AC2A1 = 6 BC2A1 , 6 EF A 6 ABC 6 CDE , respe tively.
=

whi h gives 6 BCA = 6 BAC . The equality 6 BAC = 6 CBA follows by symme-
=

try. E F
J4. Answer: 10 . A C A C A C A C
By Viete's theorem, the possible values of x y are in luded in the set of roots
of the quadrati equation E F
+

F E
a2 10a + 20 = 0 : D B D F
B D B D E
B
This equation has two dierent roots be ause D 2  > . Viete's (a) (b) ( ) (d)
formulae give to be the sum of these roots. It remains to he k that is not
= 10 4 20 0

10 11

among the roots (as y x y


x y
from the rst equation, x y 6 enables
=
+
+ = 11 Figure 2
us to nd the orresponding values for x and y ).
11 ( + )

Consider ase (d). Sin e EF A and ADE are opposite angles of a y li quadri-
J5. Answer: a) , and ; ) .
1 2 8 5
lateral ADEF and 6 ADE 6 ADC 6 CDE 6 ABC 6 CDE , we have
= + = +

a), b) Observe that if m is odd, then both m and m are even. Hen e if 6 EF A = 180Æ 6 ADE = 180Æ 6 ABC 6 CDE :
K k is odd, then k K , and k and k are not both odd. This gives the p
following three possibilities.
= = 2
S2. Answer: 3 3 .
1) If both k and k are even, then k k  k  K . Let a b+ 6 K. By the AM-GM inequality we have
2) If k is odd and k is even, then k k K .
= 2 = 4 = 8 +

k
3) If k is even and k is odd, then k k  k K .
 a + b + 3  3 K2
= 3 = (2 3) = 4 3
 K = 2 = 2 ( 3) = 2 (2 3) = 4 6
ab 6 6 K K :
The numbers K , K and K are pairwise distin t sin e modulo they
=
3 3 27

are ongruent to , and , respe tively. For a), K gives k 2 f ; ; g .


8 4 3 4 6 4

Hen e if K 6 , or equivalently K 6 p , the required ondition is satis-


0 1 2 = 1 1 2 8 2
) Let now K be even. If k is even, then we get the same three possibilities for 1 3 3

k as above. If k is odd, then k is even and k an be either even or odd.


ed. However, if K > p and a b K , then a b K and
27

4) If k is even, then k k k K K .


5) If k is odd, then k k K .
= 2 = 4 = 4( 3) = 4 12 3 3 = = = + + =

k K
3

K2
> K , so the ondition is not satised.
= 3 = 2 3 = 2( 3) 3 = 2 9

Sin e K is even, the numbers K , K , K and K are ongruent to ab K 


=

, , and , respe tively, modulo . Moreover, K is ongruent to either


8 4 3 4 6 4 12
27

or modulo . Hen e these ve numbers are pairwise distin t. S3. We use indu tion on n .
0 5 2 4 8 2 9

3 7 8

S1. Angle EF A is equal to either 6 ABC 6 CDE , or 6 ABC 6 CDE , or The proposition holds for n sin e is divisible by 0 and is not
divisible by .
Base: = 0 1 3 = 1
1
6 CDE 6 ABC , or Æ 6 ABC 6 CDE .
Answer: +
3 = 3
180

Given the hords AB and CD , the hord EF an be drawn in four essentially Step:Observe the equality
dierent ways  point E an lie on the ir le between points D and A , between : : : 1} = 11 : : : 1}  : : : 0} 1 00 : : : 0} 1 :
points A and C , between points C and B or between points B and D (see 11
| {z | {z 1 00
| {z | {z
Fig. 2). 3 +1
n 3n 3n 1 3n 1
Let us nd 6 EF A for ase ( ). 6 Sin e EF6 C and CDE are 6 angles subtended The rst fa tor here is divisibleby but not by +1 by the indu tion hypothesis,
n n

by the same hord EC , we have EF C CDE ; similarly CF A 6 ABC . and the se ond fa tor is divisible by+1 but not by . +2Sin e is prime, this implies
3 3

Hen e that the produ t is divisible by , but not by .


= = 3 9 3
n n
3 3

6 EF A = 6 CF A + 6 EF C = 6 ABC + 6 CDE : S4. Answer: . 2012

3 4
Denote a1 =and a2 q . It is easy to see that
p = Spring Open Contest: Mar h 2001
a p if k ; ; ; : : :;
a q if k ; ; ; : : :;
k = = 1 7 13

a q p if k ; ; ; : : :;
k = = 2 8 14 Juniors (up to 10th grade)

p if k ; ; ; : : :;
k = = 3 9 15

a k =
q if k ; ; ; : : :;
= 4 10 16
1. Eight students, Anne, Mary, Cathy and Tina, Anthony, Mark, Carl and Tom have
a
a p q if k ; ; ; : : :.
k = = 5 11 17
to work in four pairs, one boy and one girl in ea h pair. They know ea h other,
k = = 6 12 18
with only these ex eptions: Anthony knows neither Anne nor Mary; Mark doesn't
Observe that the sum of any six onse utive members of the sequen e is equal to know Mary and both Carl and Tom know neither Cathy nor Tina. How many
zero. Denoting S a1 : : : a , we get ways are there to divide the students into pairs, so that ea h boy ould work with
a girl he knows?
k = + +

S p if k ; ; ; : : :;
k

S p q if k ; ; ; : : :;
k = = 1 7 13

S q if k ; ; ; : : :;
k = + = 2 8 14
2. In a triangle ABC , the lengths of the sides are onse utive integers and median
q p if k ; ; ; : : :;
k = 2 = 3 9 15
drawn from A is perpendi ular to the bise tor drawn from B . Find the lengths
S
S q p if k ; ; ; : : :;
k = 2 = 4 10 16
of the sides of triangle ABC .
if k ; ; ; : : :.
k = = 5 11 17

S k = 0 = 6 12 18
3. In a s hool lo ker room there are lo kers
Hen e q p S1997 and q p S2002 , whi h give q and in three rows. The lo kers in ea h row are la-
60
K
with S2000 p q . belled from left to right with numbers to
= = 2002 2 = = 1997 = 5

p L
in the top row, to in the middle row and M
= 2007 = + = 2012 1 20

to in the bottom row.


21 40
n2 2
S5. for even n and n for odd n . 1 41 60

Kate's, Lisa's and Mary's lo kers are lo ated as shown in the gure. Ea h of the
Answer:

Divide the points into two subsets with ardinalities as lose to ea h other as
4 4

possible, and draw a line segment between any two points from dierent subsets. three lo ker numbers is divisible by the number of Mary's house, whi h is not . 1

Then ea h losed line formed by these line segments ontains an even number a) What is the number of Mary's house?
of links and hen e the line segments do not form any triangles2 with verti es at b) What ould be the numbers on the girls' lo kers?
 n 2
the given points. The number of line segments is n
for even n and 4. Integers a , b , and d satisfy ja bdj jad b j . Prove that either
jaj jbj or j j jdj .
= + = + = 1
2 4
2
n
n n
for odd n .
= = 1 = = 1
1 + 1 1

5. A onvex hexagon is onstru ted from n pie es, ea h of whi h


=

Now prove that there annot be more line segments. Consider any olle tion of line
2 2 4

segments satisfying the onditions of the problem. Let m be the maximal number is an equilateral triangle (one example is given in the gure).
of line segments in ident to one point, and let X be any point in ident to m line a) Prove that the hexagon is equiangular.
segments. Let A be the set of the other endpoints of these m line segments, and b) Find all possible values of n .
B be the set of the other n m points (in luding X ). Ea h point of A an be
joined only to points of B be ause any two joined points from A together with
X would form a triangle. Hen e ea h of the m points of A o urs as an endpoint Seniors (11th and 12th grade)
for at most n m line segments. On the other hand, ea h of the n m points
from B o urs as an endpoint for at most m line segments by the hoi e of m . 1. The serial numbers of lottery ti kets are -digit integers. It is known that the
So there is at most m n m n m m m n m segment-endpoint pairs, serial number of a winning ti ket has seven distin t digits and is divisible by ea h
7

and sin e every line segment has two endpoints, we have at most m n m line
( )+( ) = 2 ( )

segments. It remains to noti e that this expression a hieves its maximum when
( )
of its digits.
the dieren e of m and n m is as small as possible, i.e. if m n m for even a) Prove that the serial numbers of all winning ti kets onsist of the same digits.
n and if jm n m j for odd n . b) Find the largest possible serial number of a winning ti ket.
=

( ) = 1

5 6
2. Let us all a onvex hexagon ABCDEF if 6 A 6 C 6 E 6 B 6 D 6 F . From the gure we obtain L K and M L K . Sin e
K , L and M are all divisible by the number of Mary's house n , the dieren es
boring + + = + + = + 21 = + 14 = + 35

a) Is every y li hexagon boring? and M L are also divisible by n . It follows that the only
b) Is every boring hexagon y li ? L K
possible value of n is . Now, sin e 6 K 6 , we obtain K or K ,
= 21 = 14

3. Find all real-valued fun tions f x dened for all real numbers whi h satisfy the and hen e L and M or L and M .
7 1 20 = 7 = 14

ondition f x f  x2 for ea h real x .


( ) = 28 = 42 = 35 = 49

2001 + (0) = 2001


J4. If the numbers a bd and ad b have the same sign, then a bd ad b
and a bd ad b a b d . Hen e a b or d . If the numbers
+ + + = +

4. For some < x; y <  , two of the three expressions 2 x 2 y , 2 x y a bd and ad b have distin t signs, then
0 = + = ( )( ) = =

and have equal values and the third one is dierent.


0 sin + sin sin ( + )
a bd ad b a b d
and a b or d . In both ases jaj jbj or j j jdj . If jaj jbj , then
+ + 0 = + + + = ( + )( + )
1

a) Whi h of the three expressions has a dierent value? ja bdj is divisible by jaj , therefore jaj jbj . In ase j j jdj , we
= = = = =

b) Give an example of x and y for whi h su h a situation o urs. similarly obtain j j jdj .
1 = +

= = 1
= = 1 =

5. There are small boxes numbered from to , and one large box. John puts J5. b) all integers n > .
some balls in some (or all) of the small boxes, and starts relo ating them by the
10 1 10
Answer: 6

following rules: a) Let a vertex of the hexagon beÆ the vertex of k triangles. Then the interior
 during ea h move, John removes all balls from any small box numbered n
angle at this vertexÆ must be k  . Sin e the interior angles of a onvex hexagon
are less thanÆ , noneÆ of them an be larger than Æ . Sin e the sum of the
60

where the number of the balls equals n ; angles is  , it follows that all the angles are equal to Æ .
180 120

 he adds these balls into boxes to n (one ball into ea h box) and puts
720 = 6 120 120

the remaining ball into the large box.


1 1
b) There must be at least pie es, be ause there is at least one triangle on ea h
side of the hexagon, and sin e the hexagon has no angles equal to Æ , ea h
6

He ontinues this way until he annot make another move a ording to these rules. triangle an lie on only one side of the hexagon.
60

Find the largest possible total number of balls in the small boxes at the beginning
of the game, for whi h it is possible to put all balls in the large box by the end of
the game.
Solutions of Spring Open Contest
n=6 n=7 n=8
J1. Answer : . 4 Figure 3
It is lear that Carl and Tom an only work with Anne and Mary: we obtain two The onstru tions for n , n ja n are given in Figure 3. We an
ways to form two pairs. Now, Anthony and Mark have to work with Cathy and ontinue the same way, adding more large triangles in the middle.
= 6 = 7 = 8

Tina, whom they both know: there are also two ways to form the two remaining
pairs. Hen e, altogether there are four ways to form the pairs. S1. Answer: b) .
9867312

J2. , and . a) Obviously, the serial number annot ontain and must be even. Now, we
annot have among the digits, be ause every even number divisible by ends
Answer: 2 3 4 0

Let D be the midpointof BC , then the medianis AD . Sin e the bise tor of 6 B is with a . If the serial number didn't have as one of its digits, it would ontain
5 5

also an altitude in the triangle ABD , that triangle is equilateral, i.e. jBDj jBAj and should be divisible by , but the sum of the remaining seven digits is ,
0 9

and in the original triangle ABC we have jBC j jABj . Sin e the lengths of
=

ontradi tion. Hen e is one of the digits. Now, the serial number is divisible by
3 3 31

the sides of triangle ABC are onse utive integers, the dieren e jBC j jABj is
= 2

and the sum of its digits is between and . The only possible sum is
9

either or . In the rst ase jABj , jBC j and the length of the side and the serial number onsists of the digits ; ; ; ; ; ; .
9 32 39 36

AC must be either or , whi h is impossible. In the se ond ase we obtain


1 2 = 1 = 2
1 2 3 6 7 8 9

jAB j , jBC j and jAC j .


0 3
b) Any -digit number onsisting of these digits is divisible by , , and .
Now we must nd the largest possible even number among these whi h is divisible
= 2 = 4 = 3 7 1 3 6 9

J3. Answer: a) ; b) ; ; or ; ; .
7 7 28 42 14 35 49 by 7 . This number is
8 = 56 . 9867312

7 8
S2. Answer: a) yes, b) no. Sin e  x sin

x and
 
x
= os 

x sin
3
x , then

= sin

+
 
= os

in both2 ases 2 2 x 2 2 y 2 x 2 x .
2 2 2

B sin + sin = sin + os = 1

A ? Let x y x y . We shall prove that both of these are equal to .


Applying the formula for x y and squaring, we get
sin + sin = sin ( + ) 1

sin( + )

C sin
2 x + sin2 y = sin2 x os2 y + sin2 y os2 x + 2 sin x sin y os x os y ;

D sin
2 x (1 os
2y) + sin2 y (1 os
2x) 2 sin x sin y os x os y = 0 ;
2 x sin2 y x sin y os x os y = 0 :
E F 2 sin 2 sin

Figure 4 Figure 5 Sin e < x; y <  , neither x nor y equals . Hen e we must have
, i.e. x y and x y equals to either
0 sin sin 0

x y x y
or , when e . So the only expression that an have a dierent
sin sin os os = 0 os( + ) = 0 sin( + ) 1

a) Let ABCDEF be a y li hexagon. Sin e the quadrilaterals ABDF , CDF B 2x y


and EF BD are also y li (see Fig. 4), we obtain value is 2 x y .
1 sin ( + ) = 1

sin ( + )

6 BDF =  6 A; 6 DF B =  6 C; 6 F BD =  6 E: b) Taking < x <  and y x  , we get


0 = +

Now,  6 A  6 C  6 E  , and hen e 6 A 6 C 6 E  .


2 2

Thus 6 B 6 D 6 F    6 A 6 C 6 E , whi h proves that the 2 x + sin2 y = sin2 x + sin2 x + 


 
( ) + ( ) + ( ) = + + = 2
2 x + os2x = 1 :
hexagon ABCDEF is boring.
+ + = 4 2 = 2 = + + sin = sin
2

b) Let us ompress a regular hexagon along its two opposite sides (see Fig. 5). Sin e  < y < x y  x <  , we have 2 x y 6 .
The new hexagon is boring sin e all its angles are equal, but it is not y li sin e + = +2
3
sin ( + ) = 1

three of its verti es lie on one ir le and the rest on another ir le. 2 2 2

x2 2 S5. .
S3. and f x x .
Answer: 41

fx
( 2001)
We shall rst show that the total number of balls in the small boxes annot ex eed
. John an empty box number only on e, sin e no balls are put into it during
Answer. ( ) = ( ) =
2001 2001

the relo ations. He an also empty box only on e, sin e at most one ball is put
41 10

Sin e for any real number y there is an x y f su h that y = x f ,


2001
(0)
= 2001 + (0)
into it (from the tenth box). Also, boxes , and an be emptied only on e.
9

2
Box number an be emptied at most twi e (at most balls will be added to it
8 7 6

the equality f y  y f
( ) = 2001 holds for any real y . Taking y we
2001
(0)
= 0
from boxes with bigger numbers). Box an also be emptied at most twi e, box
5 5

at most times, box at most times and box at most times. John an
4
2
f
get f (0) = and hen e f
(0)
or f (0) = 0. Therefore the fun tion
(0) = 2001
therefore put no more than balls in the large box.
3 4 2

41
7 1 21

We shall now nd a way to pla e balls in the smallboxes, so that all boxes ould
2001
2 2
must be either f y y or f y y
( ) = . It is easy to he k that both
( ) =
( 2001)
be emptied. No balls are added to box , therefore it must ontain balls. One
41

of these satisfy the given onditions. ball will be added to box , therefore it must ontain balls in the beginning.
2001 2001 10 10

Similarly there must be , and balls in boxes , and respe tively. Sin e
9 8

S4. a) 2x y an be the only expression with a dierent value; b) balls are added to box , it must ontain balls in the beginning in order to
6 4 2 8 7 6

be emptied twi e. Box must ontain one ball, box three balls, boxes and
Answer: sin ( + ) 5 5 5

hoose < x < arbitrarily and take y x .


0
2

= +
2 must ontain ball. The number of balls in the small boxes is now . It is easy
4 3 2 1

a) Let 2 x y . We shall prove that in this ase also 2 x 2 y . to he k that if John always empties the box with the smallest possible number,
1 41

all balls will nally be in the large box.


sin ( + ) = 1 sin + sin = 1

From the equation 2 x y we have either x y  or x y  .


sin ( + ) = 1 + =
2
+ =
3

9 10
Final Round of National Olympiad: Mar h 2001 (1) A is a word;
(2) if w is a word, then ww and ww are also words, where w is obtained from
w by repla ing all letters A with B and all letters B with A ( xy denotes the
9th grade
on atenation of x and y );
1. John had to solve a math problem in the lass. While leaning the bla kboard, he (3) all words are reated by rules (1) and (2).
a identally erased a part of his problem as well: the text that remained on board Prove that any two words with the same number of letters dier exa tly in half of
was  x  , where  marks an erased digit. Show that John an their letters.
still solve his problem, knowing that x is an integer.
37 (72 + 3 ) = 14 45

2. Dividing a three-digit number by the number obtained from it by swapping its 11th grade
rst and last digit we get as the quotient and the sum of digits of the original
number as the remainder. Find all three-digit numbers with this property.
3

1. The angles of a onvex n -gon are ; ; : : : ; n . Find all possible values of n


and the orresponding values of .
2

3. A ir le of radius is tangent to two adja ent sides of a square and interse ts


its two remaining sides at the endpoints of a diameter of the ir le. Find the side
10

length of the square. 2. A student wrote a orre t addition operation BA DC FE to the bla kboard, su h
+ =

4. It is known that the equation jx j jx j : : : jx j a has exa tly that both summands are irredu ible fra tions and F is the least ommon multiple
one solution. Find a . of B and D . After that, the student redu ed the obtained sum EF orre tly by
1 + 2 + + 2001 =

5. A table onsisting of rows and olumnsis lled with integers ; ; : : :; an integer d . Prove that d is a ommon divisor of B and D .
in su h a way that ea h of these integers o urs in the table exa tly times and
9 2001 1 2 2001

the integers in any olumn dier by no more than . Find the maximum possible
9
3. Points D , E and F are taken on the sides BC , CA , AB of a triangle ABC ,
value of the minimal olumn sum (sum of the numbers in one olumn).
3
respe tively, so that the segments AD , BE and CF have a ommon point O .
Prove that jjOD AOj jAE j jAF j
j jEC j jF B j .
= +

10th grade 4. Let2 2 x 2and 2y be non-negative real numbers su h that x y . Prove that
xy x y 6 .
+ = 2

1. A onvex n -gon has exa tly three obtuse interior angles. Find all possible values
( + ) 2

of n . 5. ConsiderÆall trapezoids in a oordinate plane with interior angles of Æ , Æ , Æ


and , su h that their bases are parallel to one of the oordinate axes and
90 90 45

2. Find the minimum value of n su h that, among any n integers, there are three all verti es have integer oordinates. Dene the of su h a trapezoid as the
135

whose sum is divisible by . total number of points with integer oordinates inside and on the boundary of the
size

trapezoid.
3

3. There are three squares in the pi ture. Find the sum of D


a) How many pairwise non- ongruent su h trapezoids of size are there?
angles ADC and BDC . b) Find all positive integers not greater than that do not appear as sizes of
2001

any su h trapezoid.
50

A B C
4. We all a triple of positive integers a; b;
( ) harmoni if a b . Prove that,
1
+
1
=
1

12th grade
for any given positive integer , the number of harmoni triples a; b; is equal
to the number of positive divisors of 2 .
( )

1. Solve the system of equations


5. A tribe alled Ababab uses only letters A and B, and they reate words a ording 
x y :
to the following rules:
sin =

sin y = x

11 12
2. Find the maximum value of k for whi h one an hoose k integers out of 3) If , then 6 a b 6 363 whi h implies a > , a ontradi -
; ; : : :; n so that none of the hosen integers is divisible by any other hosen tion.
= 3 300 32 = 200 + 7 10

integer.
1 2 2

p
9-3. .
3. Let I and r be the midpoint and radius of the in ir le of a right-angled triangle
Answer: 10 + 5 2

ABC with the right angle at C . Rays AI and BI interse t the sides BC and
AC at points D and E , respe tively. Prove that 6B C
jAE j jBDj r .
1 1 1
+ =

4. Prove that, for any integer a > , there is a prime p su h that a a2 : : : a 1 p


O
q

is omposite.
1 1+ + + +

5. Consider a  table, lled with real numbers in su h a way that ea h number -


A
in the table is equal to the absolute value of the dieren e of the sum of numbers
3 3

in its row and the sum of numbers in its olumn. Figure 6


a) Prove that any number in this table an be expressed as a sum or a dieren e Introdu e a oordinate system where the sides of the square tangent to the ir le
of some two numbers in the table. are on the oordinate axesthen the entre of the ir le is O ; (see Fig. 6).
b) Show that there exists su h a table with numbers in it not all equal to 0. Let the side of the square be a (evidently a > ) and the interse tion points of
(10 10)

the ir le with its two other sides be A and B . As AB is the diameter of the
10

ir le, the ommon pointp C a; a of these two sides lies on the ir le. Sin e
p CO
is a radius,pwe obtain a 2 a 2 , giving a
( )
Solutions of Final Round and
.
( 10) + ( 10) = 10 10 = 5 2

a
9-1. x = 1271 .
= 10 + 5 2

9-4. .
Answer:

From the given equality we obtain x) =  . To nd the number


y = 24 + x , note that
Answer: 1001000

Note that if x is a solution of the equation, x is also a solution. For


111(24 + 14 45

uniqueness we have x x , or x . In this ase


2002

 < 1445 < 222000 = 111  2000 ;


= 2002 = 1001
111 1000 = 111000

a : : : + 2 + 1 + 0 + 1 + 2 + : : : + 999 + 1000 =
therefore y is a 4-digit number, with as its rst digit. Evidently y must end = 1000 + 999 +

with . Let y ab , where 6 a; b 6 . Writing out the multipli ation we


1
= (1000 + 1) + (999 + 2) + : : : + (2 + 999) + (1 + 1000) =

see that b ends with , hen e b and there is a arry of at least from
5 = 1 5 0 9
= 1000  1001 = 1001000 :

the third position. Sin e there is no arry to the rst position, we have a 6 .
+ 5 4 = 9 1

If the arry from the third position were more than , we would have a > , a
2

ontradi tion. Hen e a and x .


1 8
Although this is not required in the problem, it an be veried that
is indeed the only solution of the equation for a .
Remark.
= 2 = 1295 24 = 1271
x = 1001 = 1001000

9-2. Answer: 441 and . 882


9-5. 24.
We look for a number ab su h that ab ba a b , or a b . Answer:

The numbers 1 an be in the same olumn only with numbers 2, 3 and 4. As there
Hen e 6 6 , and we have 3 ases.
= 3 +( + + ) 32 = 100 + 7

1) If , then 6 a
3

whi h implies 6 a 6 . If a , are altogether  of these, the 1-s an be at most in four olumns. If all 1-s are
b6 in the same olumn, the minimal olumn sum is 9. If the 1-s are in two olumns,
4 9

then b and b . If a , then b and b is not an


= 1 100 32 = 100 + 7 163 4 5 = 4

integer.
128 = 100 + 7 = 4 = 5 160 = 100 + 7
one of these must ontain at least 5 of them and the sum of this olumn is at most
2) If , then 6 a whi h implies 6 a 6 . If a ,   . If the 1-s are in four olumns, then the sum of all numbers in
these olumns is , hen e the minimal olumn sum is at most
5 1 + 4 4 = 21
b6 
then b and b is not an integer. If a , then b , giving
= 2 200 32 = 200 + 7 263 7 8 = 7 9 (1 + 2 + 3 + 4) = 90
h i
b . = 8
224 = 200 + 7 = 8 256 = 200 + 7 90

4
. If the 1-s are in three olumns, we should have 3-s and 4-s in these
= 22

13 14
olumns to obtain the largest olumn sum. In this ase the sum of numbers in the right-angled triangles, we have
three olumns is  and the minimal olumn sum is at most 24.  
From the table below we see that this value is indeed attainable.
9 (1 + 3 + 4) = 72
6 ADF = 6 ADC 6 F DG = 6 ADC 6 DF G =

1 1 1 2 2 6 7 ... 2001
2
 
6 ADC 6 BDC ;
1 1 1 2 2 6 7 ... 2001 =

1 1 1 2 2 6 7 ... 2001
2

3 3 3 2 2 6 7 ... 2001 that implies 6 ADC 6 BDC  6 ADF . The segments AF and DF are
3 3 3 2 5 6 7 ... 2001
+ = +

transformed into ea h other by a Æ rotation around F . Hen e AF D is an


2

3 3 3 5 5 6 7 ... 2001 90

4 4 4 5 5 6 7 ... 2001 isos eles right-angled triangle with 6 ADF , yielding 6 ADC 6 BDC .
 3

4 4 4 5 5 6 7 ... 2001
= + =
4 4

4 4 4 5 5 6 7 ... 2001 Remark: There are also solutions using the osine theorem or the identity
10-1. Answer: The possible values of n are , and . 4 5 6
tan(
+

) =


tan
. + tan

The sum of the angles of a n -gon is n   . Sin e three of these angles are 1 tan tan

10-4. As a and b are non-zero integers, we have


( 2)

greater than and less than  , and the remaining n angles are greater than
2
3

0and less or equal to  , we obtain n    < n   < n     .


2
( 3) 0 + 3
2
( 2) ( 3)
2
+3
1

a
+
1

b
=() a ab
1+b
=
1


() ab = (a + b) ()
Dividing by  and transforming yields < n < . As n is an integer, we have
7
7 () ab a b = 0 () ab a b + 2 = 2 ()
6 n 6 , and it is easy to he k that all these three values are indeed possible.
4 6
2
() (a )(b ) = 2:
10-2. Answer: n . = 5 Now let a b . If a and b are positive, then a > and b > . On
1
+
1
=
1
0 0

The sum of any three integers ongruent to , and modulo is divisible the other hand, if a > and b > , then a and b are positive. Hen e the
by . Also, the sum of any three integers ongruent to ea h other modulo is
0 1 2 3

harmoni triples a; b; are 2in one-to-one orresponden e with pairs of positive


0 0

divisible by . Consequently, among any ve numbers there are three whose sum
3 3

integers r; s , where rs 2, and there are as many su h harmoni triples as


( )

is divisible by . On the other hand, among the numbers , , and there are
3

there are positive divisors of .


( ) =

no three with a sum divisible by .


3 1 3 4 6

10-5. We use indu tion on the length of a word. Let u1 and u2 be any dierent words
D of the same length, and suppose the laim holds for all shorter words. As there is
only one word of length 1, u1 and u2 are onstru ted by rule (2). This implies
that there exist words v1 and v2 so that u1 v1v1 or u1 v1v1 and u2 v2v2
B or u2 v2v2 . Note that v1 and v2 are of the same length. If v1 v2 v , then
= = =

A C
one of the words u1 and u2 is vv and the other vv , diering exa tly in half of
= = =

their letters. If v1 6 v2 , then v1 and v2 dier exa tly in half of their letters by
the indu tion hypothesis, and it remains to show that the latter halves of u1 and
=
F G
u2 also dier exa tly in half of their letters. If these halves are v1 and v2 or v1
Figure 7 and v2 , this is obviously true. The words v1 and v2 , as well as v2 and v1 , dier
exa tly in the letters where v1 and v2 oin idediering therefore also exa tly
10-3. Answer:
3
.  in half of their letters. Hen e in any ase u1 and u2 dier in half of their letters.
Consider points F and G as shown on Fig. 7. As BCD and DGF are ongruent
4
11-1. Answer: The only possibilities are n ,  and n ,  . = 3 =
6
= 4 =
5

15 16
Obviously n > . As the sum of angles of the n -gon is n  n  n ,
3
( + )
= ( 2)
11-4. Denote = 1 x , then x = 1 and from x + y=2 we get y = 1+ . Now
x2 y2 (x2 + y2 ) )2 (1 + )2  ((1 )2 + (1 + )2 ) =
2

we have n nn . Be ause of onvexity, we have n nn < 


=
2 (

( + 1)
2)
=
2 (

+1
2) =

=
(1

((1 )(1 + ))2  (2 + 2 2) =


yielding n < . If n , we obtain  ; if n , then  .
5 = 3 =
6
= 4 =
5
= 2(1 2)2 (1 + 2) = 2(1 4)(1 2) :

11-2. Let D0 and B0 0be the0 multipliers of the0 rst and the se ond fra tion, respe tively. Sin e x; y > ,4we have2 j j 6 that implies 6 2 6 and 6 4 6 .
Hen e 6 .
0 1 0 1 1 0 1 1

Then E AD B C and F BD DB0 , with B0 and D0 oprime sin e F


2(1 )(1 ) 2

is the least ommon multiple of the denominators. If, for a prime p , p divides d
= + = =

with k >0 , then p divides both E and0 F . Suppose p does not 0divide B . From0
k
11-5. Answer: a) ; b) , , , , , , , , ja .
7 1 2 3 4 6 8 10 16 28 32
k

F BD we obtain that p divides D , hen e p also divides B C E AD .


0
k
Consider a trapezoid of height h and the length of its shorter base a (see Fig. 9).
Therefore, p divides0 either B0 or C , and as B0 and D0 are oprime, p divides
= =
The longer base of the trapezoid is of length a h and thus there is a total of +

C . From F DB we get that D is divisible by p , hen e p is a ommon fa tor


=
k
N (a; h) = (a +1) + (a +2) + : : : + (a + h +1) =
(2 + a h +2)(h +1)
of C and D , ontradi ting the irredu ibility of DC . We on lude that p divides k 2

B , and similarly also D . Sin e this is true for any prime divisor p of d , then B points with integral oordinates inside and on the border of this trapezoid.
and D are both divisible by d .
11-3. Draw a line parallel to BC through A and denote its interse tion points with 6
rays BE and CF by L and M , respe tively (see Fig. 8). From similar triangles q q q
a q q q q q q

jAE j jALj . Analogously jAF j jAM j . Moreover,


q q q q q q q q q q

AEL and CEB we have


q q q q q q q q q q

jEC j jBC j =
jF B j jBC j = q
h q q q q q q q q

j AOj jALj
q q q q q q q q q q

from similar triangles AOL and DOB we get jODj jBDj , and analogously =
q

q
q

q
q

q
q

q
q

q
q

q
q

q
q

q
q

q
q

q -
jAOj jAM j Figure 9
jODj jDC j . Hen e
=

jAOj jALj jAM j jALj jAM j jALj jAM j jAE j jAF j a) We have to nd the number of distin t pairs a; h for whi h N a; h .
Taking into a ount that   , we onsider two ases:
( ) ( ) = 2001

jBC j jBC j jEC j jF B j :


+ +

jODj jBDj jDC j jBC j


2001 = 3 23 29

1) If h k is even, then N a; h a k  k where k > and


= = = + = +
+
= 2 ( ) = ( + + 1) (2 + 1) 2 +1 3

C
a k
+ >k
+ 1 >
k
+ 1. The fa tor k an be , or , yielding the
2 + 1
2 + 1 3 23 29

pairs ; , ; and ; .
2
(665 2) (75 22) (54 28)

D 2) If h k is odd, then N a; h a k  k , where h N a; h


k > and a k > k . The fa tor k an be , ,
= 2 1 ( ) = (2 +2 + 1) ( )
L a
or , yielding the pairs ; , ; , ; and ; .
1 2 +2 +1 2 +3 1 3 23 1 2 + 3
O 29 (999 1)
a (330 5) (20 45) (5 57)
E 2 3 + 6

F b) For h ; ; ; : : :; we express the size of a trapezoid in a


terms of a (see the table); if h > , then N a; h > for any
= 1 2 3 7 3 4 + 10

a
a > . It is easy to he k that numbers , , , , , , ,
A B 7 ( ) 50 4 5 + 15

a
, and are the only ones that annot be expressed by
1 1 2 3 4 6 8 10
5 6 + 21
M
any of the formulae in the table. a
16 28 32
6 7 + 28

Figure 8 12-1. Answer: The only solution is x y . a


= = 0
7 8 + 36

17 18
Clearly x y is a solution. We know that j xj 6 jxj , where equality holds 12-4. If a , then p gives the desired result:
i x (this an be easily proved using derivatives). Now
= = 0 sin = 2 = 11

: : : + 210 = 211
= 0
1+ 2+4+ 1 = 2047 = 23  :
89
jxj > j xj = jyj > j sin yj = jxj ;
sin
If a > , then a > and there exists a prime2 p that divides a . Hen e a
and at least one of the inequalities is stri t if x 6 or y 6 . is ongruent to modulo p and M a a : : : a 1 is divisible by p .
2 1 1 1
p

We also have M > a > p , implying that M is omposite.


= 0 = 0 1 p = 1+ + + +

12-2. Answer: n.
p 1+ p

Let the hosen integers be a1; : : : ; a and, for ea h i ; : : :; k , let n be the 12-5. a) Let r1 , r2 , r3 be the sums of numbers in the rst, se ond and third row, and
exponent of in the prime fa torization of a , i.e. a  b with b odd. Sin e 1 , 2 , 3 be the sums of numbers in the rst, se ond and third olumn. Denote by
k = 1 i
ni

6 b 6 n , there are n possibilities for the numbers b . If k > n , then a the element in the i -th row and j -th olumn, and noti e that all the elements
2 i i = 2 i i

there exist indi es i and j su h that b b b and n > n . Then a of the table are non-negative.
1 i 2 1 i + 1 ij
b ni

is divisible by a  b. Sin e r1 r2 r3 1 2 3 , we have


i = j = i j i = 2
nj
j = 2

If k 6 n , then hoose any k numbers in the set f n ; : : :; n g . None of them


+ + = + +

is divisible by another sin e n <  n . jr1 1j j r2 r3 2 3 j j r2 2 r3 3)j =


+ 1 2

2 2 ( + 1)
a11 = = ( + ) ( + ) = ( ) +(

jr2 2j  jr3 3 j a22  a33 :


12-3. Let 6 IAE 6 BAI and 6 DBI 6 IBA , then 6 EIA 6 BID = =

(see Fig. 10). Applying the sine rule for triangle AEI and the equality As all the elements are non-negative, a22 and a33 annot both have minus sign
= = = = = = +

r jAI j , we obtain here and, onsequently, a11 is equal to the sum or dieren e of two numbers in
the table. The proof for all other elements of the table is similar.
= sin

jAE j jAI j r
sin( + )
=
sin 6 AEI
=
sin sin 6 AEI
: b) The tables below satisfy the required ondition for any real x > : 0

B x x x x
;
0 0

x 0 x x x x :
0 x 0 2x 2x 2x

D I IMO Team Sele tion Test: April 2001


r First Day
 
C E A 1. Consider on the oordinate plane all re tangles whose
Figure 10 (i) verti es have integer oordinates;
(ii) edges are parallel to oordinate axes;
From triangle BDI , we similarly get (iii) area is , where k ; ; : : :.
2
k
= 0 1 2

Is it possible to olor all points with integer oordinates in two olors so that no
jBDj jBI j = r
: su h re tangle has all its verti es of the same olor?
+ ) sin 6 IDB sin sin 6 IDB
=
sin(
2. Point X is taken inside a regular n -gon of side length a . Let h1; h2; : : : ; h be
Sin e sin 6 AEI = os and sin 6 IDB = os , we have the distan es from X to the lines dened by the sides of the n -gon. Prove that
n

sin os sin os sin( + ) 


: :::+ > :
1 1 1 1 1 1 2

jAE j jBDj + = +
r sin( + ) r sin( + )
=
r sin( + )
=
r h1
+
h2
+
hn a

19 20
3. Let k be a xed real number. Find all fun tions f R ! R su h that 3. If k then f x x or f x ; if k 6 then f x k or
f (x) = 0 .
: Answer: = 1 ( ) = ( ) = 0 = 1 ( ) = 1

f (x) + f (y) 2 = kf (x + y2 )

Substituting y = 0 in the original equation we get
for all real numbers x and y . k
( 1) ( f x) = f (0)2 : (1)
If k 6 , then substituting x in (1) we get f or f k .
So the solutions in the ase k 6 are tthe onstant fun tions f x and
= 1 = 0 (0) = 0 (0) = 1
Se ond Day
fx k .
= 1 ( ) = 0

4. Consider all produ ts by 2,o 4, 6, ..., 2000 of the elements of the set
( ) = 1

If k , then from (1)2 we get f


= 1 . Substituting x in the original(0) = 0 = 0

A
n 1 1 1
; ; ; : : :; ;
1
. Find the sum of all these produ ts.
1
equation we get f y f y2 , and furhter substituting y we nd that
or f .
= ( ) = ( ) = 1
2 3 4 2000 2001
f(1) = 1 (1) = 0

5. Find the exponent of in the representation of the number | : :{z:: : : } as For any non-negativereal number z there is a real number y su h that y2 z , =

therefore from f y 2 f y2 we get that f z > for any z > . Also,


37 111 11

3372000 digits
produ t of prime powers.
( ) = ( ) ( ) 0 0

substituting x y in the original2 equation we get f y2 2f y 2 , so


2 = ( ) = ( )

6. Let C1 and C2 be the in ir le and0 the ir um ir le of the triangle ABC , respe - f z 6 for any z 6 . Sin e f y f y2 f y 2 f y , then we
must have f y f y , i.e. f is an odd fun tion. p
( ) 0 0 ( ) = ( ) = (( ) ) = ( )

tively. Prove that, for any point A on C2 , there exist points B0 0 0 and0 C 0 su h that ( ) = ( )

C1 and C2 are the in ir le and the ir um ir le of triangle A B C , respe tively. Now let x be any real number and z > , then denoting z y we get 0 =

f x z f x y2 f x f y 2 f x f y2 f x f z :
( + ) = ( +

) = ( )+ ( ) (2) = ( )+ ( ) = ( )+ ( )

Solutions of Sele tion Test


Hen e if a 6 b , then f b f a b a f a f b a > f a , i.e. f is
non-de reasing.
( ) = ( + ( )) = ( ) + ( ) ( )

1. Answer: Yes. Sin e f is an odd fun tion, (2) holds also when x and z are both negative. Now
Color the points with integer oordinates in three olors so that on ea h diagonal we show, using indu tion on n , that f nx nf x for any real x and integer n .
y x k all points are of the same olor and the olors hange in a y li manner
Indeed, this holds for n and if f nx nf x then
( ) = ( )

when k in reases. Sin e  for even m and  for odd


= +
m m = 0 ( ) = ( )

m , it is easy to understand that all three olors are present in verti es of ea h


2 1 (mod 3) 2 2 (mod 3)

f ((n +1)x) = f (nx + x) = f (nx)+ f (x) = nf (x)+ f (x) = (n +1)f (x) :


re tangle under onsideration. Now re oloring the verti es of some olor with one
of the remaining two olors, we obtain a oloring with the required properties. Sin e f is odd, we also have f nx f nx nf x , i.e. f nx nf x
holds for all integers n .
( ) = ( ) = ( ) ( ) = ( )

2. Let S be the area of the n -gon and r its inradius, then S n  ar . On the other =
Earlier we proved that f or f . If f then substituting
in f nx nf x we get that f n for all integers n , and sin e f is
(1) = 1 (1) = 0 (1) = 0
2
x
hand, S  a  h1 h2 : : : h . Using the AM-HM inequality, we get
=
1
( + + + n) non-de reasing, we have f x for all real x . We show now that if f
= 1 ( ) = ( ) ( ) = 0

then f x x for all x . For integers we get it from f nx nf x , substituting


( ) = 0 (1) = 1
2
( ) = ( ) = ( )

x . For a rational number we have


n a
6 h1 h2 n : : : h
+ + + n
=
2S
na
= r: = 1
b
:::+
1 1 1

h1 h2 hn
+ +
 a a
a = f (a) = f b  = bf ;
Comparing the lengths of ir umferen es of the n -gon and its in ir le, we get b b
na > r . Hen e a
2
so f b
a
=
b
. Assume that for some real number x we have f x 6 x , then ( ) =
1
+
h1 h2
+ :::+
1
> n > 2 :
hn r a
1
f (x) = x + " where " 6= 0 . If " > , then let r be a rational number su h
0

21 22
that x < r < x " , and if " < , then let r be a rational number su h that and note that
x > r > x " . In the rst ase we get r < x " f x 6 f r r , in the se ond
+ 0

37k+1 37k 37


ase r > x " f x > f r r , a ontradi tion.
+ + = ( ) ( ) =

+ = ( ) ( ) = 1000 1 = 1000 1 =
   
37k  37k + 37k 2 + : : : + 37k 36 :
4. Answer: 499 . 1001
= 1000 1 1 + 1000 1000 1000

The value of The exponent of in the representation of number 37 is k by the


2001
k

indu tion hypothesis. Hen e it su es to show that the exponent of in the
37 1000 1 + 1

     
representation of
37

1+
1

2
 1+
1

3
::: 1+
2001
1
1

37k + 37k 2 + : : : + 37k 36


is equal to the sum of all produ ts of the elements of set A by 1, 2, 3, ..., 2000, 1 + 1000 1000 1000

and the value of is . Sin e  then 37k  1 (mod 37) . Let 37k = 37q + 1 ,
then
1 1000 1 (mod 37) 1000 1000
     
1
1
 1
1
::: 1
1
1
2 3 2001
37k + 37k 2 + : : : + 37k 36=
is equal to a similar sum where the produ ts by 2, 4, 6, ..., 2000 are taken with
1 + 1000 1000 1000

q + 1) + (37q + 1)2 : : : + (37q + 1)36 


a plus sign but the produ ts by 1, 3, 5, ..., 1999 are taken with a minus sign. = 1 + (37

Denote the required sum by S , then  1 + (37q + 1) + (2  37q + 1) + : : : + (36  37q + 1) =



      =
37 36
 q
37 + 37 = 37
2  18  q + 37  37 (mod 372) :
2 S = 1+
1

2
 1+
1

3
::: 1+
2001
1
+
2

 1
 
 1

:::
 1

So 1 + 1000
37 k
+
37 2 : : :
1000
37 36 is divisible by
k
+ + 1000
k
but not by 2 , 37 37
+ 1
2
1
3
1
2001
2 =
and the exponent of in the representation of 37 2000 is k .
37
+1
1000
k
1 +2

=
3
 :::
4 2002
+
1
 :::
2 2000
2 = Hen e the exponent of in the representation of 37
37 is . 1000 1 2001

6. Let I and O be the in enter and the ir um enter of the triangle ABC , respe -
2 3 2001 2 3 2001

; tively. We know by Euler's formula that jOI j2 R2 Rr , where r and R are


2002 1 1
= + 2 = 999

the radii of the in ir le and the ir um ir le, respe tively.


2 2001 2001 = 2

and S 1001
. Assume now that there exists a point A on the ir le C2 su h that it is impossible
0
= 499
2001 to onstru t the0 points B0 and C 0 as required 0in the problem. Let the tangents
5. . drawn from A to the ir le C1 tou h C1 0 in0 B and C 0 , hen e B0C 0 is not tan-
Answer: 2001

As and are relatively prime it is su ient to nd the exponent of in the gent to the ir le C1 . Suppose the line B C and the ir le C1 have no points in
representation of the number
37 9 37
ommon0 0(the ase of two ommon points is similar). Let the distan e between the
line B C and the ir le C1 be Æ0 > . 0 0

3372000 372000 Now start moving the points B and C along the ir le C2 towards0 0 A0 in su h
|999 : :{z
:: : : 99} = 10 1 = 1000 1 :
a way that the distan es from the ir le C1 to the straight lines A B and A0C 0
3372000 numbers remain equal (note that they are both equal to at the beginning)  denote
this distan e by  . The distan e Æ obviously de reases, whereas the distan e 
0

We show by indu tion on k that the exponent of 37 in the representation of in reases, hen e at some moment they must be ome equal. Now we0 an in rease
37
1000 is k . In the ase k we have
k
1 +1 = 0
the radius r by0 0Æ 0  > to make it the in ir le of the triangle A B0C 0 . Hen e
the triangle A B C has ir umradius R and inradius r Æ , but the distan e jOI j
= 0
370 3
1 = 999 = 3  37 ;
is the same as for the triangle ABC , hen e Euler's formula for triangle A0B0 C 0
1000 +

i.e. the exponent of is . Suppose now that for some k our assertion holds,
37 1 be omes violated.
23 24

También podría gustarte